Have a question about our books, syllabus, etc.? Ask away...
User avatar
 
noah
Thanks Received: 1192
Atticus Finch
Atticus Finch
 
Posts: 1541
Joined: February 11th, 2009
 
This post thanked 1 time.
 
 

Errata list - Manhattan LSAT 3rd edition LOGIC GAMES

by noah Thu Oct 18, 2012 9:53 am

Important ones:
p.18 (chart): "Closed Grouping" is now called "Basic Grouping"!
p.497: the game should state that at least one firefighter respond to any given floor.

Minor Typos:
p.17: "We have eight elements, and each of these has to be placed in two different rows." This should say "each of these has to be placed in one of two different rows." p.31: Diagram reads "N before both L and S" -- should read "L before both S and N"
p.18 (bottom): "evaluate" misspelled.
p.19: asproaches - not a new word in the English language!
p.27 3rd paragraph break: starts with "from this rule...", "or they" is spelled incorrectly.
p. 35: "...ahead or behind of ..." should be "...ahead of or behind..."
p.36: the first notated tip with "free radicals" does not contain the final parenthesis in the quotation.
p. 111: first paragraph should start with "N can occupy any position from two to eight, as can R."
p.162: #9, the correct answer is (D), as we say later!
p.199: "whomever" should be "whoever."
p.198: answer (D) is missing a period at the end.
p.214: "the type of wash R's car receives" -- should read "the type of wash M's car receives"
p.253: s in middle of page
p.258: "use the third frame" -- should read "use the third frame."
p.258: "With our without" -- should read "With or without"
p.498: the text says "Bettleheim motors swill" rather than "Bettleheim Motors will."
p.518 - if do you (in the final paragraph) should have read "if you do" & should read "forge ahead" not "forge forward"
p.522 (2nd paragraph, last sentence) "or maybe you are [at] a point..."
Solution for PT52, G4: In Frame or Not to Frame: This should read "Two options for rule 4, but watch the conditionals!" Remove line between M and K in frame i, and remove that line in diagram for #22
 
lookendearing
Thanks Received: 0
Forum Guests
 
Posts: 1
Joined: October 26th, 2012
 
 
 

Re: Errata list - Manhattan LSAT 3rd edition LOGIC GAMES

by lookendearing Sat Oct 27, 2012 6:40 am

I don't know if this is supposed to go here but I noticed an inconsistency between the way the term "at least" is diagrammed when it's first introduced on page 28 and then when it's diagrammed on page 35. On p.28 "L finishes at least two spots ahead of P" is diagrammed as L_+P while on p.35 "O finished at least two spots ahead of M" is diagrammed as M_ _+O or O_ _+M. Am I missing something?
User avatar
 
noah
Thanks Received: 1192
Atticus Finch
Atticus Finch
 
Posts: 1541
Joined: February 11th, 2009
 
 
 

Re: Errata list - Manhattan LSAT 3rd edition LOGIC GAMES

by noah Mon Oct 29, 2012 1:45 pm

lookendearing Wrote:I don't know if this is supposed to go here but I noticed an inconsistency between the way the term "at least" is diagrammed when it's first introduced on page 28 and then when it's diagrammed on page 35. On p.28 "L finishes at least two spots ahead of P" is diagrammed as L_+P while on p.35 "O finished at least two spots ahead of M" is diagrammed as M_ _+O or O_ _+M. Am I missing something?

Thanks for asking. The second one is actually "O finished at least two spots ahead or behind of M" -- thus you need to show both options.
 
timmydoeslsat
Thanks Received: 887
Atticus Finch
Atticus Finch
 
Posts: 1136
Joined: June 20th, 2011
 
 
trophy
Most Thanked
trophy
First Responder
 

Re: Errata list - Manhattan LSAT 3rd edition LOGIC GAMES

by timmydoeslsat Wed Oct 31, 2012 12:59 pm

Page 251: The last rule regarding Ralston in the second problem has it "Raltson." :ugeek:
 
cmcole09
Thanks Received: 0
Vinny Gambini
Vinny Gambini
 
Posts: 1
Joined: November 05th, 2012
 
 
 

Re: Errata list - Manhattan LSAT 3rd edition LOGIC GAMES

by cmcole09 Mon Nov 12, 2012 4:26 pm

On page 162 you say B is correct, but D is the true answer and you say that in the individual explanation portion.
User avatar
 
noah
Thanks Received: 1192
Atticus Finch
Atticus Finch
 
Posts: 1541
Joined: February 11th, 2009
 
 
 

Re: Errata list - Manhattan LSAT 3rd edition LOGIC GAMES

by noah Mon Nov 12, 2012 5:47 pm

cmcole09 Wrote:On page 162 you say B is correct, but D is the true answer and you say that in the individual explanation portion.

Thanks!
 
sumukh09
Thanks Received: 139
Atticus Finch
Atticus Finch
 
Posts: 327
Joined: June 03rd, 2012
 
 
trophy
Most Thanked
trophy
First Responder
 

Re: Errata list - Manhattan LSAT 3rd edition LOGIC GAMES

by sumukh09 Tue Dec 25, 2012 4:56 pm

Pg. 49

Should be a double arrow over "N" and "O" in the second frame for question 4 to indicate that their positions are interchangeable.
 
practicemakesperfect
Thanks Received: 0
Forum Guests
 
Posts: 4
Joined: September 01st, 2012
 
 
 

Re: Errata list - Manhattan LSAT 3rd edition LOGIC GAMES

by practicemakesperfect Fri Jan 04, 2013 7:51 pm

On p.27, you diagram "L finishes two spots ahead of P." as "L _ P" and say it's easy to misinterpret as "L _ _ P." Then you diagram "O finishes at least two spotss ahead of M." (p.35) as "O _ _ + M" and not "O _ + M." Is one of these a mistake? Or am I missing something? I'm really confused.
User avatar
 
ManhattanPrepLSAT1
Thanks Received: 1909
Atticus Finch
Atticus Finch
 
Posts: 2851
Joined: October 07th, 2009
 
 
 

Re: Errata list - Manhattan LSAT 3rd edition LOGIC GAMES

by ManhattanPrepLSAT1 Sat Jan 05, 2013 7:42 pm

practicemakesperfect Wrote:On p.27, you diagram "L finishes two spots ahead of P." as "L _ P" and say it's easy to misinterpret as "L _ _ P." Then you diagram "O finishes at least two spotss ahead of M." (p.35) as "O _ _ + M" and not "O _ + M." Is one of these a mistake? Or am I missing something? I'm really confused.

Hey practicemakesperfect! Those are both accurate. The issue is the difference between "exactly two spots" and "at least two spots."

In the first case "L _ P" we want to make sure that L is in the spot numbered exactly two spots ahead of P. Count forward from P and in the second spot, place L.

In the second case "O _ _ + M" we know that O is at least two spots ahead of M. Count forward from M two spots and then take note that there could be more than two spots between O and M.

Hope that helps!
User avatar
 
ManhattanPrepLSAT1
Thanks Received: 1909
Atticus Finch
Atticus Finch
 
Posts: 2851
Joined: October 07th, 2009
 
 
 

Re: Errata list - Manhattan LSAT 3rd edition LOGIC GAMES

by ManhattanPrepLSAT1 Sat Jan 05, 2013 7:45 pm

sumukh09 Wrote:Pg. 49

Should be a double arrow over "N" and "O" in the second frame for question 4 to indicate that their positions are interchangeable.

Thanks for catching that!
 
practicemakesperfect
Thanks Received: 0
Forum Guests
 
Posts: 4
Joined: September 01st, 2012
 
 
 

Re: Errata list - Manhattan LSAT 3rd edition LOGIC GAMES

by practicemakesperfect Sat Jan 05, 2013 11:15 pm

mattsherman Wrote:
practicemakesperfect Wrote:On p.27, you diagram "L finishes two spots ahead of P." as "L _ P" and say it's easy to misinterpret as "L _ _ P." Then you diagram "O finishes at least two spotss ahead of M." (p.35) as "O _ _ + M" and not "O _ + M." Is one of these a mistake? Or am I missing something? I'm really confused.

Hey practicemakesperfect! Those are both accurate. The issue is the difference between "exactly two spots" and "at least two spots."

In the first case "L _ P" we want to make sure that L is in the spot numbered exactly two spots ahead of P. Count forward from P and in the second spot, place L.

In the second case "O _ _ + M" we know that O is at least two spots ahead of M. Count forward from M two spots and then take note that there could be more than two spots between O and M.

Hope that helps!


Thank you for the explanation, Matt. I am still confused, though. I might have picked the worse of the two examples I had in mind. On p.28, you have "L finishes at least two spots ahead of P." and you diagram it as "L _ + P". Then you diagram "O finishes at least two spots ahead of M." as "O _ _ + M." Doesn't that actually say that O finishes at least 3 spots ahead of M? Suppose we have this rule too: "O finishes second." In that case, if O finishes at least two spots ahead of M, M can be forth, right? But, according to the way you diagrammed the O and M rule, M can finish fifth, sixth etc., but not forth. Does that make sense? Or am I just (being) incredibly thick and missing something really obvious? ;)
User avatar
 
ManhattanPrepLSAT1
Thanks Received: 1909
Atticus Finch
Atticus Finch
 
Posts: 2851
Joined: October 07th, 2009
 
 
 

Re: Errata list - Manhattan LSAT 3rd edition LOGIC GAMES

by ManhattanPrepLSAT1 Sat Jan 05, 2013 11:27 pm

Page 35 reads "O finishes at least two spots ahead or behind of M" and is diagrammed:
"O _ _ + M" or "M _ _ + O"
practicemakesperfect Wrote:Doesn't that actually say that O finishes at least 3 spots ahead of M?

You're absolutely right. Page 35 should be adjusted to say:
"O _ + M" or "M _ + O"

Thanks for catching that!
 
hyewonkim89
Thanks Received: 5
Atticus Finch
Atticus Finch
 
Posts: 122
Joined: December 17th, 2012
 
 
trophy
Most Thankful
trophy
First Responder
 

Re: Errata list - Manhattan LSAT 3rd edition LOGIC GAMES

by hyewonkim89 Sun Feb 03, 2013 12:46 am

Hello,

I found a small mistake on pg. 40.

In the second sentence of the second to the last paragraph, "you want to able to" should be "you want to [be] able to."
User avatar
 
daniel
Thanks Received: 0
Elle Woods
Elle Woods
 
Posts: 62
Joined: July 31st, 2012
Location: Lancaster, CA
 
 
 

Re: Errata list - Manhattan LSAT 3rd edition LOGIC GAMES

by daniel Tue Feb 26, 2013 2:42 pm

On p. 111, the text says:

Manhattan LSAT 3rd edition LOGIC GAMES Wrote:N can occupy any position from two to seven, as can R.


I believe that this should be corrected to:

N can occupy any position from two to eight, as can R.


There are 8 variables in this diagram, N and R are both strays, and either one of them can be last.
User avatar
 
daniel
Thanks Received: 0
Elle Woods
Elle Woods
 
Posts: 62
Joined: July 31st, 2012
Location: Lancaster, CA
 
This post thanked 1 time.
 
 

Re: Errata list - Manhattan LSAT 3rd edition LOGIC GAMES

by daniel Tue Feb 26, 2013 9:16 pm

Here's another one, which confused me the first time I read it:

pg. 158 summarizes the implications resulting from the necessary conditions of rules #2 and #3 incorrectly.

The text currently says:

Manhattan LSAT 3rd edition LOGIC GAMES Wrote:We also learn a great deal about J. Since we know that in one situation the consequence is that J will come before K and L, and in the other it's that J will come after both K and L, we know there will never be a situation in this game where J will come between K and L.


According to Rule #2, if G is more expensive than H, then J is more expensive than both K and L.

According to Rule #3, if H is more expensive than G, then K is more expensive than both J and L.

So, according to rule#3, it is definitely possible for J to come between K and L. Therefore, the statement on p. 158 is incorrect.

I can't really suggest any correction, however, since what we know about J is actually not as straight forward as what the text is describing. In one frame, J can be ranked in any position from 2 to 6, whereas in the other frame J can be ranked in any position from 1 to 4.

Note, however, that the diagram at the bottom of page 159 gets it right (thank goodness!). ;)
 
sumukh09
Thanks Received: 139
Atticus Finch
Atticus Finch
 
Posts: 327
Joined: June 03rd, 2012
 
 
trophy
Most Thanked
trophy
First Responder
 

Re: Errata list - Manhattan LSAT 3rd edition LOGIC GAMES

by sumukh09 Thu Mar 28, 2013 7:23 pm

Page 246: For question number 2 the explanation for why A is incorrect says "there doesn't seem to be a reason we couldn't organize the remainder QOLKM" but that organization doesn't work since both L and M are in and L is supposed to come after M.
User avatar
 
daniel
Thanks Received: 0
Elle Woods
Elle Woods
 
Posts: 62
Joined: July 31st, 2012
Location: Lancaster, CA
 
 
 

Re: Errata list - Manhattan LSAT 3rd edition LOGIC GAMES

by daniel Mon Jun 17, 2013 1:13 pm

p. 402: In the explanation for Q2, the book states, "All that's left to do is compare (D) and (E) and test out any difference, which in this case is whether V can be in class 3."

Actually, class 3 is common to both answer choices, so the text and the diagram that follows should focus on testing whether V can appear in class 1, which is in (E) but not in (D).
 
james.h.meyers
Thanks Received: 2
Vinny Gambini
Vinny Gambini
 
Posts: 19
Joined: June 07th, 2013
 
 
 

Re: Errata list - Manhattan LSAT 3rd edition LOGIC GAMES

by james.h.meyers Sat Jul 06, 2013 4:38 pm

daniel Wrote:p. 402: In the explanation for Q2, the book states, "All that's left to do is compare (D) and (E) and test out any difference, which in this case is whether V can be in class 3."

Actually, class 3 is common to both answer choices, so the text and the diagram that follows should focus on testing whether V can appear in class 1, which is in (E) but not in (D).


I saw this too. I think the difference that is supposed to be made is that in the original question, choice D should have Class 1 and Class 2. Then the diagram and answer that follows would make sense. Sure, it can be remedied either way, but I think the smaller error makes more sense in terms of mistakes.

This is of course assuming that we aren't both missing something else and completely wrong.
User avatar
 
daniel
Thanks Received: 0
Elle Woods
Elle Woods
 
Posts: 62
Joined: July 31st, 2012
Location: Lancaster, CA
 
 
 

Re: Errata list - Manhattan LSAT 3rd edition LOGIC GAMES

by daniel Sun Jul 07, 2013 12:41 am

james.h.meyers Wrote:
daniel Wrote:p. 402: In the explanation for Q2, the book states, "All that's left to do is compare (D) and (E) and test out any difference, which in this case is whether V can be in class 3."

Actually, class 3 is common to both answer choices, so the text and the diagram that follows should focus on testing whether V can appear in class 1, which is in (E) but not in (D).


I saw this too. I think the difference that is supposed to be made is that in the original question, choice D should have Class 1 and Class 2. Then the diagram and answer that follows would make sense. Sure, it can be remedied either way, but I think the smaller error makes more sense in terms of mistakes.

This is of course assuming that we aren't both missing something else and completely wrong.


Ah, but in PT 16, S1, G1, Q2, (from LSAC) choice (D) says "class 2, class 3", and this answer choice is printed correctly in the LG Guide on pp. 395 & 401. I don't think it would be appropriate to change the game or the answer choices from what was originally published by LSAC. (IMHO, of course.)
 
james.h.meyers
Thanks Received: 2
Vinny Gambini
Vinny Gambini
 
Posts: 19
Joined: June 07th, 2013
 
 
 

Re: Errata list - Manhattan LSAT 3rd edition LOGIC GAMES

by james.h.meyers Tue Jul 09, 2013 12:37 am

daniel Wrote:
james.h.meyers Wrote:
daniel Wrote:p. 402: In the explanation for Q2, the book states, "All that's left to do is compare (D) and (E) and test out any difference, which in this case is whether V can be in class 3."

Actually, class 3 is common to both answer choices, so the text and the diagram that follows should focus on testing whether V can appear in class 1, which is in (E) but not in (D).


I saw this too. I think the difference that is supposed to be made is that in the original question, choice D should have Class 1 and Class 2. Then the diagram and answer that follows would make sense. Sure, it can be remedied either way, but I think the smaller error makes more sense in terms of mistakes.

This is of course assuming that we aren't both missing something else and completely wrong.


Ah, but in PT 16, S1, G1, Q2, (from LSAC) choice (D) says "class 2, class 3", and this answer choice is printed correctly in the LG Guide on pp. 395 & 401. I don't think it would be appropriate to change the game or the answer choices from what was originally published by LSAC. (IMHO, of course.)


Well there ya go. I didn't see that. I guess the concept remains the same, but yeah it should match the actual test.